Coordinates relative to a basis (linear algebra)

Click For Summary
The discussion revolves around finding the coordinates of the polynomial p(x) = (-2 + 0x - x^2) relative to the basis B = {-4 - x^2, -8 + 4x - 2x^2, -14 + 12x - 4x^2} in P2. The initial approach involved setting up a matrix and multiplying it by the vector representation of p(x), but the results were incorrect. A user suggested using the inverse of the matrix to find the correct coordinates, leading to confusion over the results. Ultimately, one participant resolved the issue and confirmed they had found the correct coordinates. The discussion highlights the importance of correctly applying matrix operations in linear algebra.
hocuspocus102
Messages
44
Reaction score
0

Homework Statement


The set B = {-4-x^2, -8+4x-2x^2, -14+12x-4x^2} is a basis for P2. Find the coordinates of p(x) = (-2 +0x -x^2) relative to this basis.


Homework Equations


n/a


The Attempt at a Solution


so the set would be in a matrix like this:
|-4 0 -1|
|-8 4 -2|
|-14 12 -4| and multiplying this by

|-2|
| 0|
|-1| as the vector for p(x) gives

| 9|
|18|
|32| but that's not the right answer and I thought that's how you'd do it.
 
Physics news on Phys.org
hocuspocus102 said:

Homework Statement


The set B = {-4-x^2, -8+4x-2x^2, -14+12x-4x^2} is a basis for P2. Find the coordinates of p(x) = (-2 +0x -x^2) relative to this basis.


Homework Equations


n/a


The Attempt at a Solution


so the set would be in a matrix like this:
|-4 0 -1|
|-8 4 -2|
|-14 12 -4| and multiplying this by

|-2|
| 0|
|-1| as the vector for p(x) gives

| 9|
|18|
|32| but that's not the right answer and I thought that's how you'd do it.
You're going at this the wrong way around. You want the vector c = <c1, c2, c3>T such that Ac = <-2, 0, -1>T, where A is the matrix you have above.
 
oh ok, so if multiply A inverse times <-2, 0, -1> would it be right? I tried that and got <-5/2, 1, 12> which I checked but is still the wrong answer. Did I just put the coordinates in the wrong order or something? because the inverse thing makes sense but it didn't accept my answer. thanks.
 
Those aren't the right coordinates. The coordinates I got check.
 
oh ok, I figured it out, thanks a lot!
 
Question: A clock's minute hand has length 4 and its hour hand has length 3. What is the distance between the tips at the moment when it is increasing most rapidly?(Putnam Exam Question) Answer: Making assumption that both the hands moves at constant angular velocities, the answer is ## \sqrt{7} .## But don't you think this assumption is somewhat doubtful and wrong?

Similar threads

  • · Replies 14 ·
Replies
14
Views
1K
Replies
4
Views
2K
  • · Replies 4 ·
Replies
4
Views
2K
  • · Replies 27 ·
Replies
27
Views
2K
  • · Replies 9 ·
Replies
9
Views
2K
  • · Replies 10 ·
Replies
10
Views
3K
  • · Replies 2 ·
Replies
2
Views
2K
  • · Replies 2 ·
Replies
2
Views
2K
  • · Replies 2 ·
Replies
2
Views
1K
Replies
9
Views
2K